Last visit was: 24 Apr 2024, 04:24 It is currently 24 Apr 2024, 04:24

Close
GMAT Club Daily Prep
Thank you for using the timer - this advanced tool can estimate your performance and suggest more practice questions. We have subscribed you to Daily Prep Questions via email.

Customized
for You

we will pick new questions that match your level based on your Timer History

Track
Your Progress

every week, we’ll send you an estimated GMAT score based on your performance

Practice
Pays

we will pick new questions that match your level based on your Timer History
Not interested in getting valuable practice questions and articles delivered to your email? No problem, unsubscribe here.
Close
Request Expert Reply
Confirm Cancel
SORT BY:
Date
Tags:
Show Tags
Hide Tags
User avatar
Current Student
Joined: 22 Oct 2006
Posts: 1212
Own Kudos [?]: 349 [63]
Given Kudos: 12
Schools:Chicago Booth '11
 Q50  V38
Send PM
Most Helpful Reply
RC & DI Moderator
Joined: 02 Aug 2009
Status:Math and DI Expert
Posts: 11162
Own Kudos [?]: 31880 [8]
Given Kudos: 290
Send PM
User avatar
Manager
Manager
Joined: 17 Aug 2005
Posts: 223
Own Kudos [?]: 1065 [5]
Given Kudos: 0
Location: Boston, MA
Send PM
Math Expert
Joined: 02 Sep 2009
Posts: 92901
Own Kudos [?]: 618704 [3]
Given Kudos: 81586
Send PM
Re: Ditrama is a federation made up of three autonomous regions. Korva. Mi [#permalink]
2
Kudos
1
Bookmarks
Expert Reply
terp26 wrote:
Ditrama is a federation made up of three autonomous regions. Korva. Mitro, and Guadar, Under the federal revenue-sharing plan, each region receives a share of federal revenues equal to the share of the total population of Ditrama residing in that region as shown by a yearly population survey. Last year the percentage of federal revenues Korva received for its share decreased somewhat even though the population survey on which the revenue-sharing was based showed that Korva's population had increased.

If the statements above are true, which one of the following must also have been shown by the population survey on which last year's revenue-sharing in Dirama was based?


(A) Of the three regions Korva had the smallest number of residents

(B) The population of Korva grew by a smaller percentage than it did in previous years

(C) The populations of Mitro and Guadar each increased by a percentage that exceeded the percentage by which the population of Korva increased.

(D) Of the three regions. Korva's numerical increase in population was the smallest

(E) Korva's population grew by a smaller percentage than did the population of at least one of the other two autonomous regions.


OFFICIAL EXPLANATION



The correct answer choice is (E)

The situation in Ditrama is as follows: Under the federal revenue-sharing plan, each region receives a share of federal revenues equal to the share of the total population of Ditrama residing in that region, as shown by a yearly population survey. Last year, the percentage of federal revenues Korva received for its share decreased somewhat even though the population survey on which the revenue-sharing was based showed that Korva’s population had increased. If the total population of Korva increased but at the same time they experienced a decrease in revenue allocation, the only possible solution is that the total population of Ditrama increased by more than the Korva increase. Thus, you must seek an answer that indicates that the total population increased more than Korva’s population increased. But be careful: this question is one of high difficulty, and the test makers do not make it easy to spot the correct answer.

Answer choice (A): Either Mitro or Guadar could have a smaller number of residents than Korva.

Answer choice (B): This answer is impossible to prove because we do not have information about the population growth of Korva in the years prior to the last one.

Answer choice (C): This is the most popular wrong answer choice. The key error is the claim that “Mitro and Guadar each increased by a percentage that exceeded” Korva’s increase. Although it must be true that at least one exceeded Korva’s increase, it does not have to be true that both exceeded Korva, as shown by the following example:

    Before After (Last Year)
    Total Population of Ditrama 30 (100%) 100 (100%)
    Population of Korva 10 (33%) 15 (15%)
    Population of Mitro 10 (33%) 10 (10%)
    Population of Guadar 10 (33%) 75 (75%)


In the example above, only one of the other regions had a population increase that exceeded Korva; the other did not. Hence this answer choice is incorrect. Note also that this example disproves answer choice (A) as well.

Answer choice (D): As shown by the previous example, this answer is incorrect.

Answer choice (E): This is the correct answer. From the stimulus we know that Korva had a population increase, but a revenue drop. So, the total population of Ditrama must have increased by more than Korva’s increase, and for this to happen, at least one other country must have had an increase in population that exceeded Korva’s. Note that the scenario in answer choice (C) would force answer choice (E) to be correct, and based on the Uniqueness Rule of Answer Choices, answer (C) is incorrect for that reason alone.
General Discussion
GMAT Instructor
Joined: 04 Jul 2006
Posts: 960
Own Kudos [?]: 693 [0]
Given Kudos: 6
Location: Madrid
 Q51  V50
Send PM
Re: Ditrama is a federation made up of three autonomous regions. Korva. Mi [#permalink]
E. Clearly its population rose less than the overall average and at least one of the other regions show a greater percentage increase in population
User avatar
VP
VP
Joined: 29 Mar 2007
Posts: 1150
Own Kudos [?]: 1737 [0]
Given Kudos: 0
Send PM
Re: Ditrama is a federation made up of three autonomous regions. Korva. Mi [#permalink]
eyunni wrote:
Ditrama is a federation made up of three autonomous regions: Korva, Mitro, and Guadar. Under the federal revenue-sharing plan, each region receives a share of federal revenues equal to the share of the total population of Ditrama residing in that region as shown by a yearly population survey. Last year the percentage of federal revenues Korva received for its share decreased somewhat even though the population survey on which the revenue-sharing was based showed that Korva’s population had increased.

If the statements above are true, which one of the following must also have been shown by the population survey on which last year’s revenue-sharing in Ditrama was based?

(A) Of the three regions, Korva had the smallest number of residents.
(B) The population of Korva grew by a smaller percentage than it did in previous years.
(C) The populations of Mitro and Guadar each increased by a percentage that exceeded the percentage by which the population of Korva increased.
(D) Of the three regions, Korva’s numerical increase in population was the smallest.
(E) Korva’s population grew by a smaller percentage than did the population of at least one of the other two autonomous regions.

Please explain your answers.


we can easily rule out AB as discussed earlier.

Essentially what MUST BE TRUE?
C: Can be true, but again does not have to be true. Perhaps only Mitro grew by a large amount. This large amount could dwarf any increase made by Korva. Thus, less money would be available for Korva. One other region must increase greater than Korva for the paradox to be solved, but two regions is not a requirement.


D: numerical pop. increased the smallest for Korva.

M and G have 100 people K has 1. Numerical increase for M and G is 5. for K its 1. Can see why % went down. So Can be true.

Here is where it can get tricky, and D becomes more of a math problem than a CR. Lets say M and G have 50 and 45 people again (respectively). K has 5 this time.

Total amount of people is 100. K is 5% of the total. Now lets say we increase M and G by 6 and K by 5. We now have 56+51+10 --> 117

10/117 is def bigger than 5%. b/c 10/200 is 5%. Thus, D does not have to be true. Numerical increase can double the amount in K, thus increasing its % of the total population by a few % points. (its now aprox 8%).
D does not finish the paradox.

E: Must be true. If no other population increased in size greater than K, then the argument would be false.

Lets go back to the numerical example again:

G, M, K. 50, 45, 5. If K increases by 5, but M increase by 4 and G increase by 4. then K should have more funds.

10/113 is def. greather than 5/100.

Same with one being static: 50, 45, 5. K increases by 5, M increases by 4. 10/104> 5/100.

One more example: K: 90. M: 5, G: 5. K increases by 5. M increase by 1. 95+5+6---> 106. 6/106. M's overall % increases slightly. K's overall % decreases slightly. This extreme example shows that one other MUST have a % increase greater than that of K regardless of the number of people in each region.

You can try any number of possibilites. If the other region does not have a population % increase greater than that of K, then the argument will crumble.

This is why E is correct.


This was a very hard CR, took me about 3 1/2-4min to solve. I honestly suggest going through it in detail as I did. I feel I have a much better understanding of how %'s and #'s work in CRs.
avatar
Intern
Intern
Joined: 31 Jul 2012
Posts: 1
Own Kudos [?]: 5 [0]
Given Kudos: 16
Send PM
Re: Ditrama is a federation made up of three autonomous regions. Korva. Mi [#permalink]
Hi everyone. I know this is an old post and that the OE is E, but i still have some doubts. Please explain.

What about if our analysis goes this way (Check the attachment please)

Attachment:
Ditrama.png
Ditrama.png [ 6.59 KiB | Viewed 18515 times ]



It would prove that if just one of the other regions grows by a greater percentage than Korva did, Korva would not always have a decreased share of population. Please your help.

Thanks in advanced
Attachments

Ditrama.xlsx [9.13 KiB]
Downloaded 140 times

avatar
Intern
Intern
Joined: 21 Oct 2012
Posts: 27
Own Kudos [?]: 63 [3]
Given Kudos: 19
Location: United States
Concentration: Marketing, Operations
GMAT 1: 650 Q44 V35
GMAT 2: 600 Q47 V26
GMAT 3: 660 Q43 V38
GPA: 3.6
WE:Information Technology (Computer Software)
Send PM
Re: Ditrama is a federation made up of three autonomous regions. Korva. Mi [#permalink]
3
Kudos
betofx wrote:
Hi everyone. I know this is an old post and that the OE is E, but i still have some doubts. Please explain.

What about if our analysis goes this way (Check the attachment please)

Attachment:
Ditrama.png



It would prove that if just one of the other regions grows by a greater percentage than Korva did, Korva would not always have a decreased share of population. Please your help.

Thanks in advanced


Hope this helps, i found it on another forum and found it useful

Argument states that although K's population increased their revenue went down, which means K's population increase was less than the COMBINED increase of M and G

Let's start with 2010 (last/base year)
2010 - Total population 100
K - 40 (40% of total)
M - 40 (40% of total)
G - 20 (20% of total)

Inference 1 : In order for K's population to increase but also for its percentage to decrease total population of Ditrama had to have increased
Inference 2 : For the sake of proving E, there are 3 possible scenarios. K has a smaller increase than none, at least 1, or both other countries

2011 Total population 110 - smaller increase than one other country
K - 43 (7.5% increase; 39.09% of total)
M - 46 (15% increase; 41.82% of total)
G - 21 (5%% increase; 19.09% of total)
2011 Total population 110 - smaller increase than both countries
K - 41 (2.5% increase; 37.27% of total)
M - 46 (15% increase; 41.82% of total)
G - 23 (15% increase; 20.91% of total)

You can see that in both scenarios K's population increased while decreasing their share of the revenue pool (% of total population)

Now let's say that K had a bigger increase than both countries:
2011 Total population 110
K - 46 (15% increase; 41.82% of total)
M - 43 (7.5% increase; 39.09% of total)
G - 21 (5% increase; 19.09% of total)

In this scenario, K increased their population but also increased their share of the revenue pool (% of total population) which cannot be true according to the argument stated

I know this is a very simplified example but it seems like whatever number you plug in with accordance to the restrictions in the argument, it's impossible for K's population increase (%-wise) to be bigger than BOTH the other countries.

Hope this wasn't too confusing and if anyone finds anything wrong with my reasoning please inform!
Intern
Intern
Joined: 26 Sep 2012
Posts: 8
Own Kudos [?]: 8 [1]
Given Kudos: 7
Location: United States
GMAT Date: 06-27-2014
GPA: 3.2
Send PM
Re: Ditrama is a federation made up of three autonomous regions. Korva. Mi [#permalink]
1
Kudos
The key here is which of the following must be true..
C is an answer which cannot must be true option, even one of the federation population % can give you the required answer. and hence E provides best answer.
User avatar
Senior Manager
Senior Manager
Joined: 05 Nov 2012
Posts: 343
Own Kudos [?]: 4585 [2]
Given Kudos: 606
Concentration: Technology, Other
Send PM
Re: Ditrama is a federation made up of three autonomous regions. Korva. Mi [#permalink]
1
Kudos
1
Bookmarks
I tried to solve it without using any calculation. Please share your strategy to solve similar problem without any calculation..

Ditrama is a federation made up of three autonomous regions. Korva. Mitro, and Guadar.
Under the federal revenue-sharing plan, each region receives a share of federal revenues equal to the share of the total population of Ditrama residing in that region as shown by a yearly population survey.
Last year the percentage of federal revenues Korva received for its share decreased somewhat even though the population survey on which the revenue-sharing was based showed that Korva's population had increased.

Paradox:
Argument says that though K's population increased, the money allotted to the region decreased.
As per premise:
money allotment to a region is proportional to the population of region. This means that the population growth in one or more region was greater than that of K.

If the statements above are true, which one of the following must also have been shown by the population survey on which last year's revenue-sharing in Dirama was based?

(A) Of the three regions Korva had the smallest number of residents
>> Not necessary. K can have second highest growth also.
(B) The population of Korva grew by a smaller percentage than it did in previous years
>> Not necessary; previous year doesn't matter . Also, it doesn't compare the growth with other regions. so avoid it.
(C) The populations of Mitro and Guadar each increased by a percentage that exceeded the percentage by which the population of Korva increased.
>> Like A, not a correct answer.
(D) Of the three regions. Korva's numerical increase in population was the smallest.
>> Like A, not a correct answer.

(E) Korva's population grew by a smaller percentage than did the population of at least one of the other two autonomous regions.
Manager
Manager
Joined: 28 Jul 2016
Posts: 123
Own Kudos [?]: 40 [0]
Given Kudos: 42
Send PM
Re: Ditrama is a federation made up of three autonomous regions. Korva. Mi [#permalink]
betofx wrote:
Hi everyone. I know this is an old post and that the OE is E, but i still have some doubts. Please explain.

What about if our analysis goes this way (Check the attachment please)

Attachment:
Ditrama.png



It would prove that if just one of the other regions grows by a greater percentage than Korva did, Korva would not always have a decreased share of population. Please your help.

Thanks in advanced


This is exactly what answer E says:
(E) Korva's population grew by a smaller percentage than did the population of at least one of the other two autonomous regions.
Korva's population increased. But in a smaller percentage than at least one of the other regions
Director
Director
Joined: 26 Oct 2016
Posts: 510
Own Kudos [?]: 3378 [0]
Given Kudos: 877
Location: United States
Concentration: Marketing, International Business
Schools: HBS '19
GMAT 1: 770 Q51 V44
GPA: 4
WE:Education (Education)
Send PM
Re: Ditrama is a federation made up of three autonomous regions. Korva. Mi [#permalink]
This is an inference question. The right answer to an inference question is something that must be true based on one or more statements in the passage. Here, we can make a deduction, and predict the correct answer.

We know that Korva's revenue share decreased even though their population increased. This must mean that, collectively, the other two countries experienced a greater proportional increase in population. Now, a key thing to understand is that in order for the other two countries to have experienced a collective increase, it doesn't have to be the case that both of these countries did. But at least one of them would have had to. Thus, choice E is correct.
Intern
Intern
Joined: 20 Aug 2020
Posts: 39
Own Kudos [?]: 8 [0]
Given Kudos: 3152
Send PM
Re: Ditrama is a federation made up of three autonomous regions. Korva. Mi [#permalink]
Sir,
For the following scenario how is option E valid?

K 100 90.1 139 90.85
M 5 4.5 7 4.57
G 6 5.4 7 4.57
VP
VP
Joined: 11 Aug 2020
Posts: 1262
Own Kudos [?]: 201 [0]
Given Kudos: 332
Send PM
Re: Ditrama is a federation made up of three autonomous regions. Korva. Mi [#permalink]
C is a trap. It may or may not be true. D must be true, else there's no way it's population increased, yet it received less revenue.

Ditrama is a federation made up of three autonomous regions. Korva. Mitro, and Guadar, Under the federal revenue-sharing plan, each region receives a share of federal revenues equal to the share of the total population of Ditrama residing in that region as shown by a yearly population survey. Last year the percentage of federal revenues Korva received for its share decreased somewhat even though the population survey on which the revenue-sharing was based showed that Korva's population had increased.

If the statements above are true, which one of the following must also have been shown by the population survey on which last year's revenue-sharing in Dirama was based?


(A) Of the three regions Korva had the smallest number of residents
-doesn't have to be true

(B) The population of Korva grew by a smaller percentage than it did in previous years
-doesn't have to be true

(C) The populations of Mitro and Guadar each increased by a percentage that exceeded the percentage by which the population of Korva increased.
-TRAP

(D) Of the three regions. Korva's numerical increase in population was the smallest
-maybe, maybe not

(E) Korva's population grew by a smaller percentage than did the population of at least one of the other two autonomous regions.
CORRECT
Intern
Intern
Joined: 09 Aug 2022
Posts: 4
Own Kudos [?]: 0 [0]
Given Kudos: 86
Send PM
Re: Ditrama is a federation made up of three autonomous regions. Korva. Mi [#permalink]
If only one region's population percentage increased but the others decreased(the %), wouldn't that make E insufficient to resolve the paradox?
On the other hand if both increased it would definitely decrease the percentage of federal revenue recieved by Korva.

Would appreciate if someone could evaluate my reasoning for choosing C over E?
GMAT Club Bot
Re: Ditrama is a federation made up of three autonomous regions. Korva. Mi [#permalink]
Moderators:
GMAT Club Verbal Expert
6917 posts
GMAT Club Verbal Expert
238 posts
CR Forum Moderator
832 posts

Powered by phpBB © phpBB Group | Emoji artwork provided by EmojiOne